1998 AJHSME Problems/Problem 23

Revision as of 12:22, 27 June 2011 by Mrdavid445 (talk | contribs) (Created page with "If the pattern in the diagram continues, what fraction of the interior would be shaded in the eighth triangle? [http://www.artofproblemsolving.com/Forum/viewtopic.php?p=2331778&...")
(diff) ← Older revision | Latest revision (diff) | Newer revision → (diff)

If the pattern in the diagram continues, what fraction of the interior would be shaded in the eighth triangle?

picture

$\text{(A)}\ \frac{3}{8}\qquad\text{(B)}\ \frac{5}{27}\qquad\text{(C)}\ \frac{7}{16}\qquad\text{(D)}\ \frac{9}{16}\qquad\text{(E)}\ \frac{11}{45}$